Difference between revisions of "2002 AMC 12B Problems/Problem 15"

m (Solution 2 - Longer but might make more sense)
(Solution 2)
 
(8 intermediate revisions by 6 users not shown)
Line 12: Line 12:
  
  
== Solution 2 - Longer but might make more sense ==
+
== Solution ==
 
Since N is a four digit number, assume WLOG that <math>N = 1000a + 100b + 10c + d</math>, where a is the thousands digit, b is the hundreds digit, c is the tens digit, and d is the ones digit.
 
Since N is a four digit number, assume WLOG that <math>N = 1000a + 100b + 10c + d</math>, where a is the thousands digit, b is the hundreds digit, c is the tens digit, and d is the ones digit.
 
Then, <math>\frac{1}{9}N = 100b + 10c + d</math>, so <math>N = 900b + 90c + 9d</math>
 
Then, <math>\frac{1}{9}N = 100b + 10c + d</math>, so <math>N = 900b + 90c + 9d</math>
Line 31: Line 31:
 
The upper limit for the right hand side (RHS) is <math>999</math> (when <math>b = 9</math>, <math>c = 9</math>, and <math>d = 9</math>).
 
The upper limit for the right hand side (RHS) is <math>999</math> (when <math>b = 9</math>, <math>c = 9</math>, and <math>d = 9</math>).
 
It's easy to prove that for an <math>a</math> there is only one combination of <math>b, c,</math> and <math>d</math> that can make the equation equal. Just think about the RHS as a three digit number <math>bcd</math>. There's one and only one way to create every three digit number with a certain combination of digits.
 
It's easy to prove that for an <math>a</math> there is only one combination of <math>b, c,</math> and <math>d</math> that can make the equation equal. Just think about the RHS as a three digit number <math>bcd</math>. There's one and only one way to create every three digit number with a certain combination of digits.
Thus, we test for how many as are in the domain set by the RHS. Since <math>125\cdot7 = 975</math> which is the largest a value, then a can be <math>1</math> through <math>7</math>, giving us the answer of <math>\boxed {D) 7}</math>
+
Thus, we test for how many as are in the domain set by the RHS. Since <math>125\cdot7 = 875</math> which is the largest <math>a</math> value, then <math>a</math> can be <math>1</math> through <math>7</math>, giving us the answer of <math>\boxed {D) 7}</math>
  
 
IronicNinja~
 
IronicNinja~
 +
 +
==Solution 2==
 +
Let Let <math>a</math> denote the leftmost digit of <math>N</math> and let <math>x</math> denote the three-digit number obtained by removing <math>a</math>. Then <math>N=1000a+x=9x</math> and it follows that <math>1000a=8x</math>. Dividing both sides by 8 yields <math>125a=x</math>. All the values of <math>a</math> in the range 1 to 7 result in three-digit numbers, hence there are <math>\boxed{D)7}</math> values for <math>N</math>.
 +
 +
∼Glowworm
  
 
== See also ==
 
== See also ==

Latest revision as of 00:37, 22 November 2024

Problem

How many four-digit numbers $N$ have the property that the three-digit number obtained by removing the leftmost digit is one ninth of $N$?

$\mathrm{(A)}\ 4 \qquad\mathrm{(B)}\ 5 \qquad\mathrm{(C)}\ 6 \qquad\mathrm{(D)}\ 7 \qquad\mathrm{(E)}\ 8$

Solution

Let $N = \overline{abcd} = 1000a + \overline{bcd}$, such that $\frac{N}{9} = \overline{bcd}$. Then $1000a + \overline{bcd} = 9\overline{bcd} \Longrightarrow 125a = \overline{bcd}$. Since $100 \le \overline{bcd} < 1000$, from $a = 1, \ldots, 7$ we have $7$ three-digit solutions, and the answer is $\mathrm{(D)}$.


Solution 1

Since N is a four digit number, assume WLOG that $N = 1000a + 100b + 10c + d$, where a is the thousands digit, b is the hundreds digit, c is the tens digit, and d is the ones digit. Then, $\frac{1}{9}N = 100b + 10c + d$, so $N = 900b + 90c + 9d$ Set these equal to each other: \[1000a + 100b + 10c + d = 900b + 90c + 9d\] \[1000a = 800b + 80c + 8d\] \[1000a = 8(100b + 10c + d)\] Notice that $100b + 10c + d = N - 1000a$, thus: \[1000a = 8(N - 1000a)\] \[1000a = 8N - 8000a\] \[9000a = 8N\] \[N = 1125a\]

Go back to our first equation, in which we set $N = 1000a + 100b + 10c + d$, Then: \[1125a = 1000a + 100b + 10c + d\] \[125a = 100b + 10c + d\] The upper limit for the right hand side (RHS) is $999$ (when $b = 9$, $c = 9$, and $d = 9$). It's easy to prove that for an $a$ there is only one combination of $b, c,$ and $d$ that can make the equation equal. Just think about the RHS as a three digit number $bcd$. There's one and only one way to create every three digit number with a certain combination of digits. Thus, we test for how many as are in the domain set by the RHS. Since $125\cdot7 = 875$ which is the largest $a$ value, then $a$ can be $1$ through $7$, giving us the answer of $\boxed {D) 7}$

IronicNinja~

Solution 2

Let Let $a$ denote the leftmost digit of $N$ and let $x$ denote the three-digit number obtained by removing $a$. Then $N=1000a+x=9x$ and it follows that $1000a=8x$. Dividing both sides by 8 yields $125a=x$. All the values of $a$ in the range 1 to 7 result in three-digit numbers, hence there are $\boxed{D)7}$ values for $N$.

∼Glowworm

See also

2002 AMC 12B (ProblemsAnswer KeyResources)
Preceded by
Problem 14
Followed by
Problem 16
1 2 3 4 5 6 7 8 9 10 11 12 13 14 15 16 17 18 19 20 21 22 23 24 25
All AMC 12 Problems and Solutions

The problems on this page are copyrighted by the Mathematical Association of America's American Mathematics Competitions. AMC logo.png